Jump to content
Urch Forums

gmatcraze

1st Level
  • Posts

    375
  • Joined

Everything posted by gmatcraze

  1. DS problem inequality. Please help to solve this.
  2. Please refer attached doc file. Please help to solve this.
  3. guys, I need help with the CR attached. Its a GMAT Prep question .... Please explain your answer choices. Thanks
  4. If x is not equal to 0, is |x| (1) x / |x| (2) |x| > x IMO, it is B ... can someone help to confirm on the correct answer for this one. Thanks
  5. Unlike the wholesale price of raw wool, the wholesale price of raw cotton has fallen considerably in the last year. Thus, although the retail price of cotton clothing at retail clothing stores has not yet fallen, it will inevitably fall. Which of the following, if true, most seriously weakens the argument above? (A) The cost of processing raw cotton for cloth has increased during the last year. (B) The wholesale price of raw wool is typically higher than that of the same volume of raw cotton. © The operating costs of the average retail clothing store have remained constant during the last year. (D) Changes in retail prices always lag behind changes in wholesale prices. (E) The cost of harvesting raw cotton has increased in the last year. OA: Please explain the question and your answer choices too.
  6. If a and b are integers greater than zero and a/b =16, what is the value of a? (1) a is a multiple of 8. (2) b is a multiple of 2. OA:
  7. John and Ed work for the same Sales Company. John receives a $600 basic salary plus 20 percent of any profit he realizes for the company in excess of $5,000, while Ed receives an $800 basic salary in addition to 40 percent of any profit he realizes for the company only if the total profit he realizes is greater than $8,000, otherwise he receives a flat salary of $1,000. For what amount of profit will both John and Ed earn the same amount? (A)$1,000 (B)$1,200 ©$2,400 (D)$6,000 (E)$7,000 Sorry, don't have the OA to this .... Please show how to solve this problem.
  8. A certain mayor has proposed a fee of five dollars per day on private vehicles entering the city, claiming that the fee will alleviate the city’s traffic congestion. The mayor reasons that, since the fee will exceed the cost of round-trip bus fare from many nearby points, many people will switch from using their cars to using the bus. Which of the following statements, if true, provides the best evidence that the mayor’s reasoning is flawed? (A) Projected increases in the price of gasoline will increase the cost of taking a private vehicle into the city. (B) The cost of parking fees already makes it considerably more expensive for most people to take a private vehicle into the city than to take a bus. © Most of the people currently riding the bus do not own private vehicles. (D) Many commuters opposing the mayor’s plan have indicated that they would rather endure traffic congestion than pay a five-dollar-per day fee. (E) During the average workday, private vehicles owned and operated by people living within the city account for twenty percent of the city’s traffic congestion. OA: Please explain your answer choice and why the other options are incorrect. Thanks
  9. Some people have questioned the judge’s objectivity in cases of sex discrimination against women. But the record shows that in sixty percent of such cases, the judge has decided in favor of the women. This record demonstrates that the judge has not discriminated against women in cases of sex discrimination against women. The argument above is flawed in that it ignores the possibility that A. a large number of the judge’s cases arose out of allegations of sex discrimination against women B. many judges find it difficult to be objective in cases of sex discrimination against women C. the judge is biased against women defendants or plaintiffs in cases that do not involve sex discrimination D. the majority of the cases of sex discrimination against women that have reached the judge’s court have been appealed from a lower court E. the evidence shows that the women should have won in more than sixty percent of the judge’s cases involving sex discrimination against women OA:
  10. A recent spate of launching and operating mishaps with television satellites led to a corresponding surge in claims against companies underwriting satellite insurance. As a result, insurance premiums shot up, making satellites more expensive to launch and operate. This, in turn, has added to the pressure to squeeze more performance out of currently operating satellites. Which of the following, if true, taken together with the information above, best supports the conclusion that the cost of television satellites will continue to increase? A. Since the risk to insurers of satellites is spread over relatively few units, insurance premiums are necessarily very high. B. When satellites reach orbit and then fail, the causes of failure are generally impossible to pinpoint with confidence. C. The greater the performance demands placed on satellites, the more frequently those satellites break down. D. Most satellites are produced in such small numbers that no economies of scale can be realized. E. Since many satellites are built by unwieldy international consortia, inefficiencies are inevitable OA:
  11. IS the standard deviation of a set less than 25000 a) range of set is 15000 b) median is 120000 Sorry, don't have the OA to this one. IMO, it shoule be E since we do not know the elements in the set. Range and Median is not sufficient to determine the SD.
  12. Guess, you are right .... thanks for the clarification ...
  13. The least common chronic medical condition in the U.S. in 1990 was carpal tunnel syndrome, and the second and third least common were athlete’s foot and ringworm, in that order. The incidence rates for both athlete’s food and ringworm decrease with age, but the incidence rate for carpal tunnel syndrome increases with age. The average age of the U.S. population will increase between 1990 and 2010. Which conclusion can be most properly drawn from the above information? (A)Carpal tunnel syndrome will be the most common chronic medical condition in 2010. (B)A higher percentage of the population will suffer from carpal tunnel syndrome in 2010 than in 1990. ©The average age of people suffering carpal tunnel will decrease between 1990 and 2010. (D)More people will suffer carpal tunnel in 2010 than in 1990. (E)In 2010, a majority of the population will have at least one of the conditions mentioned. OA: These kind of CR problems that involve percentages and numbers drive me crazy :(.... Can somebody suggest a strategy on how tot tackle such kind of problems?
  14. gmatcraze

    DS prob

    Is xy (1) x > 1 and y (2) 2x – y > 1 OA:
  15. Can the above method be solved using Alligation?
  16. Ronald: According to my analysis of the national economy, housing prices should not increase during the next six months unless interest rates drop significantly. Mark: I disagree. One year ago, when interest rates last fell significantly, housing prices did not increase at all. It can be inferred from the conversation above that Mark has interpreted Ronald’s statement to mean that (A) housing prices will rise only if interest rates fall (B) if interest rates fall, housing prices must rise © interest rates and housing prices tend to rise and fall together (D) interest rates are the only significant economic factor affecting housing prices (E) interest rates are likely to fall significantly in the next six months Can someone help with this ... OA seems to be though I fail to figure it out:(
  17. For the purpose of stimulating innovation at TechCorp, one of the company's long-standing goals has been to obtain at least 50 percent of its annual revenues from sales of products that are no more than three years old. Last year, TechCorp achieved this goal, despite the fact that the company introduced no new products during the year. Which of the following, if true, best explains the results described above? a. None of the company's competitors introduced any new products during the last year. b. Scientists at the company report that they are close to breakthroughs that should result in several new products during the coming year. c. Sales of some of the company's older products were discontinued during that last year. d. The company has introduced very few new products during the last three years. e. Company spending on research and development has increased sharply over the past five years. This has been discussed previously but somehow I fail to understand the given OA ... so let's discuss this again ... OA:
  18. Through various ethically questionable practices such as maintaining offshore bank accounts, establishing shell companies, shielding investors from investigation, and a reliance on internal account auditing, JPD Enterprises has realized unprecedented profits over the past two decades. (A)shielding investors from investigation, and a reliance on (B)shielding investors from investigation, and through a reliance on ©and shielding investors from investigation, and relying on (D)and shielding investors from investigation, and through a reliance on (E)and shielding investors from investigation, relying on Can someone help to explain the parallellism in this question. Will post the OA later. Thanks.
  19. so will (B) be correct if it was worded as follows: (B) even though it is by no means certain that the expenditure of large sums of money will produce My doubt is why the usage "produces" is preferred over "will produce"
×
×
  • Create New...